Recent content by flintbox

  1. F

    Gauss' Law between infinite plates

    I've used Gauss to determine the Electric field inside to be ##2\pi \rho x## (CGS units), but what about outside? I don't know how to apply Gauss since there is no charge enclosed.
  2. F

    Gauss' Law between infinite plates

    Then the field points outwards! Thanks
  3. F

    Gauss' Law between infinite plates

    The E field just above the center points upward and the E field below downwards. Thank you! I think I can do it now.
  4. F

    Gauss' Law between infinite plates

    But I am confused about the direction of the electric field inside the plates, since there is a charge density everywhere.
  5. F

    Gauss' Law between infinite plates

    Homework Statement The volume between two infinite plates located at x=L and x=-L respectively is filled with a uniform charge density ##\rho##. Calculate the electric field in the regions above, between and below the plates. Calculate the potential difference between the points x=-L and x=L...
  6. F

    Interpreting ##\hat{e}_z## in Maxwell's equations

    Hi, I'm trying to interpret a form of Maxwell's equations, but I can't seem to figure out where the term $\^{e}_z$ comes from in the following equation: ## \frac{\partial{\vec{E}_t}}{\partial{z}}+i\frac{\omega}{c}\hat{e}_z\times \vec{B}_t=\vec{\nabla}_tE_z ##
  7. F

    Prove Lorentz invariance for momentum 4-vector

    You're right! Thank you so much.
  8. F

    Prove Lorentz invariance for momentum 4-vector

    Homework Statement I am meant to show that the following equation is manifestly Lorentz invariant: $$\frac{dp^{\mu}}{d\tau}=\frac{q}{mc}F^{\mu\nu}p_{\nu}$$ Homework Equations I am given that ##F^{\mu\nu}## is a tensor of rank two. The Attempt at a Solution I was thinking about doing a Lorents...
  9. F

    Is Qij=AiBj a Tensor of Rank 2?

    Thanks a lot! I think I understand it now: $$A_i' B_j' = R_{in}A_n R_{jm}A_m$$ $$A_i' B_j' = R_{in} R_{jm} (A_n A_m)$$ $$A_i' B_j' = R_{in} R_{jm} Q_{nm}$$ $$A_i' B_j' = Q'_{nm}$$ So we for proving something is a tensor, we just apply some transformations to it, right?
  10. F

    Is Qij=AiBj a Tensor of Rank 2?

    Homework Statement Suppose A and B are vectors. Show that the object Q with nine components Qij=AiBj is a tensor of rank 2. Homework Equations A tensor transforms under rotations (R) as a vector: Tij'=RinRjmTnm The Attempt at a Solution I wanted to just create the matrix, but I don't know how...
Back
Top